LSAT and Law School Admissions Forum

Get expert LSAT preparation and law school admissions advice from PowerScore Test Preparation.

 Administrator
PowerScore Staff
  • PowerScore Staff
  • Posts: 8929
  • Joined: Feb 02, 2011
|
#104154
Complete Question Explanation

Except, Resolve the Paradox. The correct answer choice is (D).

Answer choice (A):

Answer choice (B):

Answer choice (C):

Answer choice (D): This is the correct answer choice.

Answer choice (E):

This explanation is still in progress. Please post any questions below!
User avatar
 aziragoramo
  • Posts: 3
  • Joined: Jan 06, 2024
|
#104709
I had a hard time eliminating answer choice B.

does patients in hospital mean admitted? Or how would you eliminate this answer choice? Thank you.
User avatar
 zoezoe6021
  • Posts: 27
  • Joined: Dec 29, 2023
|
#104873
Could you please explain why E is correct instead of D?

I made an assumption below

citizen #: X is 50, and Y is 100
patients #: X is 1, and Y is 4

I think D can explain this disproportion. Many patients from X go to Y to receive treatments.
E seems irrelevant. It's hard for me to draw a relationship between lifestyle stress and patient # without making extra assumptions.
User avatar
 srusty
PowerScore Staff
  • PowerScore Staff
  • Posts: 32
  • Joined: Nov 30, 2023
|
#104894
Hi Zoe! So in this case, our stimulus does not specify that the the residents of city X are residents in city X hospitals, and the residents of city Y are residents only in city Y hospitals. So, answer choice (D) does not explain why less residents of City X go to the hospital, regardless of where they go. Answer choice (E) assumes a correlation between stress and health levels, which helps explain why City X would have less resident hospitalizations, regardless of where they go! Hope this helps.

Get the most out of your LSAT Prep Plus subscription.

Analyze and track your performance with our Testing and Analytics Package.